Please confirm topic selection

Are you sure you want to trigger topic in your Anconeus AI algorithm?

Please confirm action

You are done for today with this topic.

Would you like to start learning session with this topic items scheduled for future?

Review Question - QID 215198

In scope icon M 7 A
QID 215198 (Type "215198" in App Search)
A 78-year-old woman presents to her primary care doctor reporting several months of shortness of breath on exertion. She finally decided to see a doctor because she had a syncopal episode while gardening yesterday. Her past medical history is notable for hypertension and hyperlipidemia, both well-controlled on atorvastatin and losartan. Her temperature is 36.9°C (98.4°F), blood pressure is 132/83 mmHg, pulse is 94/min, respirations are 18/min, and oxygen saturation is 96% on room air. On auscultation, there is a systolic murmur best heard at the upper right sternal border, as well as a soft single S2. Which of the following is the most appropriate next step in management?